10 svar
821 visningar
Zeus är nöjd med hjälpen
Zeus 604
Postad: 12 nov 2020 21:49 Redigerad: 12 nov 2020 23:03

Spänningsfall

Hej!

Är det sant att en lampa i en krets lyser p.g.a. spänningsfall, så den potentiella elektriska energin omvandlas till ljus?

D.v.s. det har inte helt med strömmen att göra.

Tack.

JohanF 4446 – Moderator
Postad: 12 nov 2020 22:36

Ja, fast i en lampa med glödtråd är det bara en bråkdel av energin som blir ljus. Den mesta energin blir värme.

En LED-lampa är bättre på att omvandla elektrisk energi till ljus.  Den blir inte lika varm.

JohanF 4446 – Moderator
Postad: 12 nov 2020 22:41

Men lampan lyser inte pga av enbart spänningsfall. Det är strömmen som får lampan att lysa. Men för att få en ström måste man ha ett spänningsfall. Dvs både ström och spänning krävs. 
P=U*I. Om antingen U eller I är noll, så blir det ingen effektutveckling.

PeterG 318
Postad: 12 nov 2020 22:43

Olika ljuskällor bygger på olika principer.

Glödlampan använder definitivt strömmen för att få en tråd att glöda som då sänder ut ljus.

Att potentiella elektriska energin genererar ljus räcker inte som förklaring.

För att kolla vidare, kika exempelvis på

http://fragelada.fysik.org/index.asp?id=723#:~:text=Lysdiod%20(Light%20Emitting%20Diode%2C%20LED,polaritet%20(n%C3%A5gra%20f%C3%A5%20volt)

Zeus 604
Postad: 12 nov 2020 22:52 Redigerad: 12 nov 2020 22:53

Jag förstår att ström också behövs, men det är väl inte strömmen i sig utan det är ju spänningsfallet som leder till att energi omvandlas.

Det är detta jag är förvirrad över. 

Enligt lösningsförslaget lyser B och C svagast. Detta förklaras med att de tillsammans har högre resistans så det flödar mindre ström genom dem. Att strömmen är mindre förstår jag. Men som sagt, spänningsfallet är det viktiga! Så frågan är, vad blir spänningsfallet i B respektive C. Spänningsfallet för de bägge är ju lika med spänningsfallet för D, så vardera har totalt ett mindre spänningsfall. D.v.s att de lyser svagare. Korrekt?

JohanF 4446 – Moderator
Postad: 12 nov 2020 22:59

Titta på formeln P=U*I så ser du vad som krävs för att få störst effektutveckling. Om den utvecklade effekten blir värme eller ljus eller något annat, beror på vilken teknik som används.

Dr. G 9351
Postad: 12 nov 2020 23:06

Spänningsfallet över B är lika stort som spänningsfallet över C (om identiska lampor), men bara hälften av spänningsfallet över D.

Zeus 604
Postad: 14 nov 2020 02:15 Redigerad: 14 nov 2020 02:17
JohanF skrev:

Titta på formeln P=U*I så ser du vad som krävs för att få störst effektutveckling. Om den utvecklade effekten blir värme eller ljus eller något annat, beror på vilken teknik som används.

Hur kommer det sig att lösningen i facit använder bara ström för att förklara vilken som lyser starkast? Deras spänningsfall U skiljer ju sig, så det borde vara viktigt att ta med i lösningen.

Låt x vara lampornas resistans. Låt R vara ersättningsresistansen för den parallella delen av kretsen. Då gäller detta:

1/R = 1/(2x) + 1/x  ——> R = 1,5x

U för de parallella delarna är således I * 1,5x

U för den ensamma lampan är I * x

Alltså skiljer sig spänningen.

Dr. G 9351
Postad: 14 nov 2020 06:59

Eftersom alla lampor har samma resistans så räcker det med att titta på strömmen (eller spänningen) för att se var effekten är störst.

P=UI=I2R=U2RP = UI = I^2R = \dfrac{U^2}{R}

Zeus 604
Postad: 14 nov 2020 11:34
Dr. G skrev:

Eftersom alla lampor har samma resistans så räcker det med att titta på strömmen (eller spänningen) för att se var effekten är störst.

P=UI=I2R=U2RP = UI = I^2R = \dfrac{U^2}{R}

Tack! Det var riktigt klokt.

JohanF 4446 – Moderator
Postad: 14 nov 2020 12:04 Redigerad: 14 nov 2020 12:19

Ja som doktorn säger, facit antar att lamporna har samma resistans, oavsett vilken ström som flyter genom den, eller spänning som ligger över den. Och för att undvika onödiga svårigheter med dina fysikkurser så ska du göra på det sättet om det inte står något annat i uppgiften. Men här är lite överkurs.

Det är en väldigt förenklad bild av resistansen i en glödlampa att säga att den är konstant. Man kan säga att den är hyfsat konstant inom små intervall av strömändringar. I själva verket så ökar resistansen markant i glödtråden ju mer ström som flyter genom den, så facit gör det ganska lätt för sig genom att hävda konstant resistans. Åtminstone när de antar  att resistansen är konstant, utan att de redovisar sitt antagande. Till exempel så är glödtråden i princip en kortslutning när strömmen närmar sig noll (kall glödtråd).

Man kommer ju naturligtvis fram till samma resultat som facit, dvs att vissa av lamporna kommer att lysa svagare än andra, även genom att ansätta en bättre, mer komplicerad modell av glödtrådens resistans, dvs en modell  där resistansen har ett strömberoende. Så för uppgiften och din gymnasiekurs spelar det ingen som helst roll, men jag tycker det hade varit mycket bättre ifall uppgiften hade formulerats med att glödlampans resistans skulle förutsättas vara konstant. Eller att uppgiften hade gjorts med resistanselement istället för glödlampor.

 

Svara Avbryt
Close